Primes of the form $((2^k-1)10^m+2^{(k-1)}+10)/42$, where m is the number of decimal digits of $2^{k-1}-1$











up vote
2
down vote

favorite
2












Primes of the form $dfrac{(2^k-1)*10^m+2^{(k-1)}+10}{42}$, where $m$ is the number of decimal digits of $2^{k-1}-1$.
With Pari I found that primes occur for $k=3,, 6, 12, 36, 105, 156,336, 2286, 4272,$ $4427, 11979, 20076, 29343, 29988, 30405$. $:$



The first thing I would ask is this: why there is only one $k=4427$ which is not a multiple of 3, whereas all the other $k$'s $(3, 6, 12, 36, 105, 156, 336, 2286, 4272, 11979, 20076, 29343, 29988, 30405)$ are congruent to $0mod3$.$:$ Is there any mathematical reason?$:$



The second question is: could be $k$ be of the form $3s+1$, i mean is there a $k$ of the form $3s+1$ such that $dfrac{(2^k-1)*10^m+2^{(k-1)}+10}{42}$ is prime?
The question is related to this other question:
A conjecture about numbers of the form $10^{m}(2^{k}−1)+2^{k-1}−1$, where $m$ is the number of decimal digits of $ 2^{k-1}$.










share|cite|improve this question









New contributor




paolo galli is a new contributor to this site. Take care in asking for clarification, commenting, and answering.
Check out our Code of Conduct.




















  • @Taneli Huuskonen@lulu is there some mathematical reason for that?
    – paolo galli
    Nov 15 at 13:10












  • (PARI) ec(n)= fromdigits(concat(digits(2^n-1), digits(2^(n-1)-1))) for(j=2, 10^4, s=ec(j); if(Mod(s+11, 42)==0, if(ispseudoprime((s+11)/42)==1, print1(j, ", "))‌​))
    – paolo galli
    Nov 15 at 13:21












  • @Especially Lime is there any particular mathematical reason why primes of this type occur more often when k is a multiple of 3?
    – paolo galli
    Nov 15 at 14:10















up vote
2
down vote

favorite
2












Primes of the form $dfrac{(2^k-1)*10^m+2^{(k-1)}+10}{42}$, where $m$ is the number of decimal digits of $2^{k-1}-1$.
With Pari I found that primes occur for $k=3,, 6, 12, 36, 105, 156,336, 2286, 4272,$ $4427, 11979, 20076, 29343, 29988, 30405$. $:$



The first thing I would ask is this: why there is only one $k=4427$ which is not a multiple of 3, whereas all the other $k$'s $(3, 6, 12, 36, 105, 156, 336, 2286, 4272, 11979, 20076, 29343, 29988, 30405)$ are congruent to $0mod3$.$:$ Is there any mathematical reason?$:$



The second question is: could be $k$ be of the form $3s+1$, i mean is there a $k$ of the form $3s+1$ such that $dfrac{(2^k-1)*10^m+2^{(k-1)}+10}{42}$ is prime?
The question is related to this other question:
A conjecture about numbers of the form $10^{m}(2^{k}−1)+2^{k-1}−1$, where $m$ is the number of decimal digits of $ 2^{k-1}$.










share|cite|improve this question









New contributor




paolo galli is a new contributor to this site. Take care in asking for clarification, commenting, and answering.
Check out our Code of Conduct.




















  • @Taneli Huuskonen@lulu is there some mathematical reason for that?
    – paolo galli
    Nov 15 at 13:10












  • (PARI) ec(n)= fromdigits(concat(digits(2^n-1), digits(2^(n-1)-1))) for(j=2, 10^4, s=ec(j); if(Mod(s+11, 42)==0, if(ispseudoprime((s+11)/42)==1, print1(j, ", "))‌​))
    – paolo galli
    Nov 15 at 13:21












  • @Especially Lime is there any particular mathematical reason why primes of this type occur more often when k is a multiple of 3?
    – paolo galli
    Nov 15 at 14:10













up vote
2
down vote

favorite
2









up vote
2
down vote

favorite
2






2





Primes of the form $dfrac{(2^k-1)*10^m+2^{(k-1)}+10}{42}$, where $m$ is the number of decimal digits of $2^{k-1}-1$.
With Pari I found that primes occur for $k=3,, 6, 12, 36, 105, 156,336, 2286, 4272,$ $4427, 11979, 20076, 29343, 29988, 30405$. $:$



The first thing I would ask is this: why there is only one $k=4427$ which is not a multiple of 3, whereas all the other $k$'s $(3, 6, 12, 36, 105, 156, 336, 2286, 4272, 11979, 20076, 29343, 29988, 30405)$ are congruent to $0mod3$.$:$ Is there any mathematical reason?$:$



The second question is: could be $k$ be of the form $3s+1$, i mean is there a $k$ of the form $3s+1$ such that $dfrac{(2^k-1)*10^m+2^{(k-1)}+10}{42}$ is prime?
The question is related to this other question:
A conjecture about numbers of the form $10^{m}(2^{k}−1)+2^{k-1}−1$, where $m$ is the number of decimal digits of $ 2^{k-1}$.










share|cite|improve this question









New contributor




paolo galli is a new contributor to this site. Take care in asking for clarification, commenting, and answering.
Check out our Code of Conduct.











Primes of the form $dfrac{(2^k-1)*10^m+2^{(k-1)}+10}{42}$, where $m$ is the number of decimal digits of $2^{k-1}-1$.
With Pari I found that primes occur for $k=3,, 6, 12, 36, 105, 156,336, 2286, 4272,$ $4427, 11979, 20076, 29343, 29988, 30405$. $:$



The first thing I would ask is this: why there is only one $k=4427$ which is not a multiple of 3, whereas all the other $k$'s $(3, 6, 12, 36, 105, 156, 336, 2286, 4272, 11979, 20076, 29343, 29988, 30405)$ are congruent to $0mod3$.$:$ Is there any mathematical reason?$:$



The second question is: could be $k$ be of the form $3s+1$, i mean is there a $k$ of the form $3s+1$ such that $dfrac{(2^k-1)*10^m+2^{(k-1)}+10}{42}$ is prime?
The question is related to this other question:
A conjecture about numbers of the form $10^{m}(2^{k}−1)+2^{k-1}−1$, where $m$ is the number of decimal digits of $ 2^{k-1}$.







number-theory






share|cite|improve this question









New contributor




paolo galli is a new contributor to this site. Take care in asking for clarification, commenting, and answering.
Check out our Code of Conduct.











share|cite|improve this question









New contributor




paolo galli is a new contributor to this site. Take care in asking for clarification, commenting, and answering.
Check out our Code of Conduct.









share|cite|improve this question




share|cite|improve this question








edited Nov 16 at 7:39





















New contributor




paolo galli is a new contributor to this site. Take care in asking for clarification, commenting, and answering.
Check out our Code of Conduct.









asked Nov 15 at 12:48









paolo galli

213




213




New contributor




paolo galli is a new contributor to this site. Take care in asking for clarification, commenting, and answering.
Check out our Code of Conduct.





New contributor





paolo galli is a new contributor to this site. Take care in asking for clarification, commenting, and answering.
Check out our Code of Conduct.






paolo galli is a new contributor to this site. Take care in asking for clarification, commenting, and answering.
Check out our Code of Conduct.












  • @Taneli Huuskonen@lulu is there some mathematical reason for that?
    – paolo galli
    Nov 15 at 13:10












  • (PARI) ec(n)= fromdigits(concat(digits(2^n-1), digits(2^(n-1)-1))) for(j=2, 10^4, s=ec(j); if(Mod(s+11, 42)==0, if(ispseudoprime((s+11)/42)==1, print1(j, ", "))‌​))
    – paolo galli
    Nov 15 at 13:21












  • @Especially Lime is there any particular mathematical reason why primes of this type occur more often when k is a multiple of 3?
    – paolo galli
    Nov 15 at 14:10


















  • @Taneli Huuskonen@lulu is there some mathematical reason for that?
    – paolo galli
    Nov 15 at 13:10












  • (PARI) ec(n)= fromdigits(concat(digits(2^n-1), digits(2^(n-1)-1))) for(j=2, 10^4, s=ec(j); if(Mod(s+11, 42)==0, if(ispseudoprime((s+11)/42)==1, print1(j, ", "))‌​))
    – paolo galli
    Nov 15 at 13:21












  • @Especially Lime is there any particular mathematical reason why primes of this type occur more often when k is a multiple of 3?
    – paolo galli
    Nov 15 at 14:10
















@Taneli Huuskonen@lulu is there some mathematical reason for that?
– paolo galli
Nov 15 at 13:10






@Taneli Huuskonen@lulu is there some mathematical reason for that?
– paolo galli
Nov 15 at 13:10














(PARI) ec(n)= fromdigits(concat(digits(2^n-1), digits(2^(n-1)-1))) for(j=2, 10^4, s=ec(j); if(Mod(s+11, 42)==0, if(ispseudoprime((s+11)/42)==1, print1(j, ", "))‌​))
– paolo galli
Nov 15 at 13:21






(PARI) ec(n)= fromdigits(concat(digits(2^n-1), digits(2^(n-1)-1))) for(j=2, 10^4, s=ec(j); if(Mod(s+11, 42)==0, if(ispseudoprime((s+11)/42)==1, print1(j, ", "))‌​))
– paolo galli
Nov 15 at 13:21














@Especially Lime is there any particular mathematical reason why primes of this type occur more often when k is a multiple of 3?
– paolo galli
Nov 15 at 14:10




@Especially Lime is there any particular mathematical reason why primes of this type occur more often when k is a multiple of 3?
– paolo galli
Nov 15 at 14:10










1 Answer
1






active

oldest

votes

















up vote
0
down vote













If 3|k then Mod(2^k - 1, 7) = 0 and Mod(2^(k-1) - 1, 7) = 3, so Mod(ec(k),7) = 3 for any m.



If 3 does not divide k, the fact that 10 is a primitive root (mod 7) comes into play. In each case, there will only be one residue class of Mod(m, 6) that makes Mod(ec(k),7) = 3.



If k == 1 (mod 3) then Mod(2^(k-1) - 1, 7) = 0 and Mod(2^k - 1, 7) = 1, so Mod(ec(k), 7) = Mod(10^m, 7). This is 3 when Mod(m, 6) = 1.



If k == 2 (mod 3) then we have Mod(ec(k), 7) = Mod(3*10^m + 1, 7), which is again 3 when Mod(m,6) = 1.



Of the 18 possible pairs (Mod(k,3), Mod(m, 6)) then, the 6 pairs with Mod(k,3) = 0 , the pair (Mod(k,3) = 1, Mod(m,6) = 1), and the pair (Mod(k,3) = 2, Mod(m,6) = 1) allow Mod(pw(k), 7) = 3.



For the remaining ten pairs, Mod(ec(k), 7) is not 3.



So, ec(k) + 4 is divisible by 7 about 6 times as often for 3|k as for k == 1 (mod 3), and about 6 times as often as for k == 2 (mod 3). A numerical check up to k = 10000 confirms this.






share|cite|improve this answer








New contributor




paolo galli is a new contributor to this site. Take care in asking for clarification, commenting, and answering.
Check out our Code of Conduct.


















    Your Answer





    StackExchange.ifUsing("editor", function () {
    return StackExchange.using("mathjaxEditing", function () {
    StackExchange.MarkdownEditor.creationCallbacks.add(function (editor, postfix) {
    StackExchange.mathjaxEditing.prepareWmdForMathJax(editor, postfix, [["$", "$"], ["\\(","\\)"]]);
    });
    });
    }, "mathjax-editing");

    StackExchange.ready(function() {
    var channelOptions = {
    tags: "".split(" "),
    id: "69"
    };
    initTagRenderer("".split(" "), "".split(" "), channelOptions);

    StackExchange.using("externalEditor", function() {
    // Have to fire editor after snippets, if snippets enabled
    if (StackExchange.settings.snippets.snippetsEnabled) {
    StackExchange.using("snippets", function() {
    createEditor();
    });
    }
    else {
    createEditor();
    }
    });

    function createEditor() {
    StackExchange.prepareEditor({
    heartbeatType: 'answer',
    convertImagesToLinks: true,
    noModals: true,
    showLowRepImageUploadWarning: true,
    reputationToPostImages: 10,
    bindNavPrevention: true,
    postfix: "",
    imageUploader: {
    brandingHtml: "Powered by u003ca class="icon-imgur-white" href="https://imgur.com/"u003eu003c/au003e",
    contentPolicyHtml: "User contributions licensed under u003ca href="https://creativecommons.org/licenses/by-sa/3.0/"u003ecc by-sa 3.0 with attribution requiredu003c/au003e u003ca href="https://stackoverflow.com/legal/content-policy"u003e(content policy)u003c/au003e",
    allowUrls: true
    },
    noCode: true, onDemand: true,
    discardSelector: ".discard-answer"
    ,immediatelyShowMarkdownHelp:true
    });


    }
    });






    paolo galli is a new contributor. Be nice, and check out our Code of Conduct.










     

    draft saved


    draft discarded


















    StackExchange.ready(
    function () {
    StackExchange.openid.initPostLogin('.new-post-login', 'https%3a%2f%2fmath.stackexchange.com%2fquestions%2f2999664%2fprimes-of-the-form-2k-110m2k-110-42-where-m-is-the-number-of-dec%23new-answer', 'question_page');
    }
    );

    Post as a guest















    Required, but never shown

























    1 Answer
    1






    active

    oldest

    votes








    1 Answer
    1






    active

    oldest

    votes









    active

    oldest

    votes






    active

    oldest

    votes








    up vote
    0
    down vote













    If 3|k then Mod(2^k - 1, 7) = 0 and Mod(2^(k-1) - 1, 7) = 3, so Mod(ec(k),7) = 3 for any m.



    If 3 does not divide k, the fact that 10 is a primitive root (mod 7) comes into play. In each case, there will only be one residue class of Mod(m, 6) that makes Mod(ec(k),7) = 3.



    If k == 1 (mod 3) then Mod(2^(k-1) - 1, 7) = 0 and Mod(2^k - 1, 7) = 1, so Mod(ec(k), 7) = Mod(10^m, 7). This is 3 when Mod(m, 6) = 1.



    If k == 2 (mod 3) then we have Mod(ec(k), 7) = Mod(3*10^m + 1, 7), which is again 3 when Mod(m,6) = 1.



    Of the 18 possible pairs (Mod(k,3), Mod(m, 6)) then, the 6 pairs with Mod(k,3) = 0 , the pair (Mod(k,3) = 1, Mod(m,6) = 1), and the pair (Mod(k,3) = 2, Mod(m,6) = 1) allow Mod(pw(k), 7) = 3.



    For the remaining ten pairs, Mod(ec(k), 7) is not 3.



    So, ec(k) + 4 is divisible by 7 about 6 times as often for 3|k as for k == 1 (mod 3), and about 6 times as often as for k == 2 (mod 3). A numerical check up to k = 10000 confirms this.






    share|cite|improve this answer








    New contributor




    paolo galli is a new contributor to this site. Take care in asking for clarification, commenting, and answering.
    Check out our Code of Conduct.






















      up vote
      0
      down vote













      If 3|k then Mod(2^k - 1, 7) = 0 and Mod(2^(k-1) - 1, 7) = 3, so Mod(ec(k),7) = 3 for any m.



      If 3 does not divide k, the fact that 10 is a primitive root (mod 7) comes into play. In each case, there will only be one residue class of Mod(m, 6) that makes Mod(ec(k),7) = 3.



      If k == 1 (mod 3) then Mod(2^(k-1) - 1, 7) = 0 and Mod(2^k - 1, 7) = 1, so Mod(ec(k), 7) = Mod(10^m, 7). This is 3 when Mod(m, 6) = 1.



      If k == 2 (mod 3) then we have Mod(ec(k), 7) = Mod(3*10^m + 1, 7), which is again 3 when Mod(m,6) = 1.



      Of the 18 possible pairs (Mod(k,3), Mod(m, 6)) then, the 6 pairs with Mod(k,3) = 0 , the pair (Mod(k,3) = 1, Mod(m,6) = 1), and the pair (Mod(k,3) = 2, Mod(m,6) = 1) allow Mod(pw(k), 7) = 3.



      For the remaining ten pairs, Mod(ec(k), 7) is not 3.



      So, ec(k) + 4 is divisible by 7 about 6 times as often for 3|k as for k == 1 (mod 3), and about 6 times as often as for k == 2 (mod 3). A numerical check up to k = 10000 confirms this.






      share|cite|improve this answer








      New contributor




      paolo galli is a new contributor to this site. Take care in asking for clarification, commenting, and answering.
      Check out our Code of Conduct.




















        up vote
        0
        down vote










        up vote
        0
        down vote









        If 3|k then Mod(2^k - 1, 7) = 0 and Mod(2^(k-1) - 1, 7) = 3, so Mod(ec(k),7) = 3 for any m.



        If 3 does not divide k, the fact that 10 is a primitive root (mod 7) comes into play. In each case, there will only be one residue class of Mod(m, 6) that makes Mod(ec(k),7) = 3.



        If k == 1 (mod 3) then Mod(2^(k-1) - 1, 7) = 0 and Mod(2^k - 1, 7) = 1, so Mod(ec(k), 7) = Mod(10^m, 7). This is 3 when Mod(m, 6) = 1.



        If k == 2 (mod 3) then we have Mod(ec(k), 7) = Mod(3*10^m + 1, 7), which is again 3 when Mod(m,6) = 1.



        Of the 18 possible pairs (Mod(k,3), Mod(m, 6)) then, the 6 pairs with Mod(k,3) = 0 , the pair (Mod(k,3) = 1, Mod(m,6) = 1), and the pair (Mod(k,3) = 2, Mod(m,6) = 1) allow Mod(pw(k), 7) = 3.



        For the remaining ten pairs, Mod(ec(k), 7) is not 3.



        So, ec(k) + 4 is divisible by 7 about 6 times as often for 3|k as for k == 1 (mod 3), and about 6 times as often as for k == 2 (mod 3). A numerical check up to k = 10000 confirms this.






        share|cite|improve this answer








        New contributor




        paolo galli is a new contributor to this site. Take care in asking for clarification, commenting, and answering.
        Check out our Code of Conduct.









        If 3|k then Mod(2^k - 1, 7) = 0 and Mod(2^(k-1) - 1, 7) = 3, so Mod(ec(k),7) = 3 for any m.



        If 3 does not divide k, the fact that 10 is a primitive root (mod 7) comes into play. In each case, there will only be one residue class of Mod(m, 6) that makes Mod(ec(k),7) = 3.



        If k == 1 (mod 3) then Mod(2^(k-1) - 1, 7) = 0 and Mod(2^k - 1, 7) = 1, so Mod(ec(k), 7) = Mod(10^m, 7). This is 3 when Mod(m, 6) = 1.



        If k == 2 (mod 3) then we have Mod(ec(k), 7) = Mod(3*10^m + 1, 7), which is again 3 when Mod(m,6) = 1.



        Of the 18 possible pairs (Mod(k,3), Mod(m, 6)) then, the 6 pairs with Mod(k,3) = 0 , the pair (Mod(k,3) = 1, Mod(m,6) = 1), and the pair (Mod(k,3) = 2, Mod(m,6) = 1) allow Mod(pw(k), 7) = 3.



        For the remaining ten pairs, Mod(ec(k), 7) is not 3.



        So, ec(k) + 4 is divisible by 7 about 6 times as often for 3|k as for k == 1 (mod 3), and about 6 times as often as for k == 2 (mod 3). A numerical check up to k = 10000 confirms this.







        share|cite|improve this answer








        New contributor




        paolo galli is a new contributor to this site. Take care in asking for clarification, commenting, and answering.
        Check out our Code of Conduct.









        share|cite|improve this answer



        share|cite|improve this answer






        New contributor




        paolo galli is a new contributor to this site. Take care in asking for clarification, commenting, and answering.
        Check out our Code of Conduct.









        answered 12 hours ago









        paolo galli

        213




        213




        New contributor




        paolo galli is a new contributor to this site. Take care in asking for clarification, commenting, and answering.
        Check out our Code of Conduct.





        New contributor





        paolo galli is a new contributor to this site. Take care in asking for clarification, commenting, and answering.
        Check out our Code of Conduct.






        paolo galli is a new contributor to this site. Take care in asking for clarification, commenting, and answering.
        Check out our Code of Conduct.






















            paolo galli is a new contributor. Be nice, and check out our Code of Conduct.










             

            draft saved


            draft discarded


















            paolo galli is a new contributor. Be nice, and check out our Code of Conduct.













            paolo galli is a new contributor. Be nice, and check out our Code of Conduct.












            paolo galli is a new contributor. Be nice, and check out our Code of Conduct.















             


            draft saved


            draft discarded














            StackExchange.ready(
            function () {
            StackExchange.openid.initPostLogin('.new-post-login', 'https%3a%2f%2fmath.stackexchange.com%2fquestions%2f2999664%2fprimes-of-the-form-2k-110m2k-110-42-where-m-is-the-number-of-dec%23new-answer', 'question_page');
            }
            );

            Post as a guest















            Required, but never shown





















































            Required, but never shown














            Required, but never shown












            Required, but never shown







            Required, but never shown

































            Required, but never shown














            Required, but never shown












            Required, but never shown







            Required, but never shown







            Popular posts from this blog

            Can a sorcerer learn a 5th-level spell early by creating spell slots using the Font of Magic feature?

            Does disintegrating a polymorphed enemy still kill it after the 2018 errata?

            A Topological Invariant for $pi_3(U(n))$